Formative Flashcards

1
Q

Question
Your client made his first appearance at the magistrates’ court yesterday charged with Actual Bodily Harm contrary to s.47 Offences Against The Person Act 1861. Your client pleaded not guilty and consented to summary trial which has been fixed for next month. The District Judge remanded your client in custody awaiting trial and your client has today appealed against that decision.
When should the appeal be heard?
Select one alternative:
* The appeal should be heard as soon as practicable and in any event no later than seven days after the original decision on bail unless the Crown Court otherwise directs
* The appeal should be heard as soon as practicable and in any event no later than 24 hours after it was served unless the Crown Court otherwise directs
* The appeal should be heard as soon as practicable and in any event no later than 48 hours after it was served unless the Crown Court otherwise directs
* The appeal should be heard as soon as practicable and in any event no later than the business day after the original decision on bail unless the Crown Court otherwise directs
* The appeal should be heard as soon as practicable and in any event no later than the business day after it was served unless the Crown Court otherwise directs

A
  • The appeal should be heard as soon as practicable and in any event no later than the business day after it was served unless the Crown Court otherwise directs

Correct. Unless the Crown Court otherwise directs, the court officer must arrange for the court to hear the application or appeal as soon as practicable and in any event no later than the business day after it was served. See Crim PR 14.8 (6). Crim PR 14.8 (7) states that the Crown Court may vary a time limit under this rule. Whilst the other answers are plausible, they are each incorrect. It is incorrect to state that the appeal should be heard no later than 24 hours after the application was served unless the Crown Court otherwise directs. This is not the correct time frame set out in the Criminal Procedure Rules. The rule refers to one business day for practical reasons. It wouldn’t be possible for the court to hear the application no later than 24 hours after it was served if, for example, it was served at 5pm on a Friday evening. It is incorrect to state that the appeal should be heard no later than 48 hours after it was served unless the Crown Court otherwise directs. This is not the correct time frame set out in the Criminal Procedure Rules. This timeframe is not practical, it would not allow for the defendant to have a timely hearing in all circumstances. It is incorrect to state that the appeal should be heard no later than the business day after the original decision on bail was made, unless the Crown Court otherwise directs. This is not the correct time frame set out in the Criminal Procedure rules. From a practical point of view, notice of the application must be served prior to the appeal hearing and it may be necessary to take instructions from the client and it is not always possible to be prepared for an appeal hearing this close to the original application. It is incorrect to state that the appeal should be heard no later than seven days after the original decision on bail unless the Crown Court otherwise directs. This is not the correct time frame set out in the Criminal Procedure Rules. The timeframe is not practical, it would not allow for the defendant to have a timely hearing in all the circumstances. Please review your notes on Bail and the procedure for appealing against bail decisions, together with the relevant Criminal Procedure Rules.

How well did you know this?
1
Not at all
2
3
4
5
Perfectly
2
Q

A woman has been arrested on suspicion of theft from a shop. The theft was of a number of shirts and jackets, to the approximate value of £1,350. A decision was made to delay her right to have someone (in this case, her boyfriend) notified of her arrest. The woman has been in detention for 12 hours. Constables have now visited the woman’s address and recovered clothing which appears to match the description of the items stolen.
Which of the following correctly sets out the position with delaying the woman’s right to inform her boyfriend of her arrest?
Select one alternative:
* It appears that the stolen goods have been recovered, the nature of the offence being theft means that it remains proportionate to delay the woman’s right to notify her boyfriend of her arrest.
* It appears that the stolen goods have been recovered, the delay is still under 36 hours, so there is no need to allow the woman to notify her boyfriend of her arrest.
* It appears that the stolen goods have been recovered, the delay is no longer necessary, and the woman should be permitted to notify her boyfriend of her arrest.
* It appears that the stolen goods have been recovered, the relatively high value of the goods means it remains proportionate to delay the woman’s right to notify her boyfriend of her arrest.
* It appears that the stolen goods have been recovered, the delay is still under 24 hours, so there is no need to allow the woman to notify her boyfriend of her arrest.

A
  • It appears that the stolen goods have been recovered, the delay is no longer necessary, and the woman should be permitted to notify her boyfriend of her arrest.

Correct. S. 56(3) PACE 1984 provides that any delay to the right to have someone informed should be proportionate and should last no longer than 36 hours. If the goods have been recovered, then the delay is no longer proportionate. Whilst the other answers are plausible, they are each incorrect. It is correct that the delay should last no longer than 36 hours and the suspect has only been in detention for 12 hours, but it is disproportionate to continue to delay her right to have someone informed of her arrest on the basis that the goods appear to have been recovered and there doesn’t appear to be any other reason to withhold her right to intimation on the facts presented. It is correct to say the delay is under 24 hours but that has no relevance to this question. The right to have someone informed can be delayed for up to 36 hours not 24 hours. The suspect has only been in detention for 12 hours but it is disproportionate to continue to delay her right to have someone informed of her arrest, on the basis that the goods appear to have been recovered and there doesn’t appear to be any other reason to withhold her right to intimation on the facts presented. It is incorrect to suggest that the suspect’s right to have someone informed of her arrest is in anyway linked to the value of goods that have been stolen. The value of the goods is not relevant to the question of whether it is proportionate to delay the woman’s right to inform someone of her arrest. It is incorrect to suggest that the suspect’s right to have someone informed of her arrest is in anyway linked to the nature of the offence that she has been arrested for. The fact that she has been arrested for theft is not relevant to the question of whether it is proportionate to delay the woman’s right to inform someone of her arrest. Please review your materials on Police Powers – the right to have someone informed of arrest and s.56 PACE 1984.

How well did you know this?
1
Not at all
2
3
4
5
Perfectly
3
Q

Question
A woman is charged with one count of causing grievous bodily harm contrary to s.20 Offences Against the Person Act 1861. She has pleaded not guilty and has applied for bail while the case is adjourned for trial. She has a number of previous convictions for violent offences committed whilst on bail. In all other respects, however, she has always complied with bail conditions in the past.
Which of the following statements best describes the court’s likely approach to bail?

Select one alternative:
* In determining whether bail should be granted, the court will consider the there is a real prospect that the woman would receive a custodial sentence, and therefore bail cannot be granted.
* In determining whether bail should be granted, the court will consider the fact that the woman has complied with bail conditions in the past and there is no real prospect that the woman would receive a custodial sentence.
* In determing whether bail should be granted, the court will consider the fact that the woman has previous convictions for violent offences as evidence to support the fact that all of the exceptions to the general right to bail exist.
* In determining whether bail should be granted, the court will consider the fact that the woman has previous convictions for violent offences to support the exception to the general right to bail that she is likely to commit further offences whilst on bail.
* In determining whether bail should be granted, the court will consider the fact that the woman has complied with bail conditions in the past and therefore none of the exceptions to the general right to bail exist.

A
  • In determining whether bail should be granted, the court will consider the fact that the woman has previous convictions for violent offences to support the exception to the general right to bail that she is likely to commit further offences whilst on bail.

Correct. The question for the court to consider is whether an exception to the general right to bail (under s.4 Bail Act 1976) should apply here. One relevant exception is whether the woman would be likely to commit further offences whilst on bail; here, the fact that she has previous convictions for doing so could be used as evidence to support that she would do so again. Whilst the other answers might be plausible, they are each incorrect. It is incorrect to state that the court will consider the fact that the woman has complied with bail conditions in the past and that this means that none of the exceptions to the general right to bail exist. Whilst the court will consider the fact that woman has complied with bail conditions in the past when making a determination on bail, an exception to the right to bail still exists on the basis that she is likely to commit further offences on bail as evidenced by her previous convictions. It is incorrect to state that the court will consider that all of the exceptions to the general right to bail exist as evidenced by the woman’s previous convictions. The previous convictions may be used as evidence to support the exception that she will commit further offences whilst on bail but they would not be evidence to support the other exceptions, such as interfering with witnesses or failing to surrender. It is incorrect to state that the court will consider the fact that the woman has complied with bail conditions in the past and therefore there is no real prospect that the woman would receive a custodial sentence in this case. The nature of the offence and her previous convictions means that a custodial sentence is likely in this case. It is incorrect to state that bail cannot be granted because there is a real prospect of the woman receiving a custodial sentence. Whilst there is a prospect of the woman receiving a custodial sentence, this is not enough to refuse bail on its own. The court will still need to consider the general right to bail and whether any exceptions apply. Please review your materials on Bail and the exceptions to the general right to bail under s.4 Bail Act 1976.

How well did you know this?
1
Not at all
2
3
4
5
Perfectly
4
Q

Question
Your client is facing trial in relation to an offence of causing grievous bodily harm with intent contrary to s.18 of the Offences Against the Person Act 1861. It is alleged that he deliberately pushed someone into the path of a cyclist. The victim fell to the ground and suffered serious head injuries. The defendant denies involvement in the offence and claims that it was another person who pushed the victim. The defendant states that this is a case of mistaken identity.
Which of the following best summarises the burden and standard of proof that applies to your client’s case?
Select one alternative:
* The burden of proving that he did not push the victim into the oncoming cyclist falls upon the defence to prove on a balance of probabilities. There is no burden on the prosecution as the offence is one of strict liability.
* The burden of proving the elements of the offence rests with the prosecution who must prove those elements beyond reasonable doubt. The defence does not have to prove anything.
* The burden of proving the elements of the offence rests upon the prosecution who must prove those elements beyond reasonable doubt, however where your client wishes to rely upon his account that he did not push the victim, the onus shifts to the defence to prove beyond reasonable doubt.
* The burden of proving the elements of the offence rests upon the prosecution who must prove those elements beyond reasonable doubt, however where your client wishes to rely upon his account that he did not push the victim, the onus shifts to the defence to prove that, on the balance of probabilities.
* The burden of proving the elements of the offence rests with the prosecution who must prove them on a balance of probabilities. The defence does not have to prove anything.

A
  • The burden of proving the elements of the offence rests with the prosecution who must prove those elements beyond reasonable doubt. The defence does not have to prove anything.

Well done, this is the single best answer as it correctly states what the prosecution has to prove and correctly states the standard of proof placed on the prosecution (beyond reasonable doubt). See Woolmington v DPP [1935] UKHL 1. The answer makes it clear that on these facts the defence does not have to prove anything. While the other answers might sound plausible, they are incorrect. There is no burden of proof on the defence in this case. It is for the prosecution to prove beyond reasonable doubt that the defendant pushed the victim. In cases where there is a burden of proof on the defendant, the standard of proof is usually on the balance of probabilities. The standard of proof for the prosecution is beyond reasonable doubt, not the balance of probabilities. The offence is not one of strict liability as s.18 has the mens rea of intention to cause GBH. Please review your materials on the burden and standard of proof as it applies in relation to the offences on the syllabus.

How well did you know this?
1
Not at all
2
3
4
5
Perfectly
5
Q

You represent a male who was arrested on 13 September at 02.20 hours and brought to the police station on suspicion of Robbery contrary to s.8 Theft Act 1968. You note that he arrived at the police station at 02.45 hours and his detention was authorised at 03.05 hours. The male would like to know when he would likely be released.
Which of the following best describes the advice you would give to the male?
Select one alternative:
* The male can be detained only until 03.05 hours on 14 September unless his detention is lawfully extended. An extension to detain until 15.05 hours on the 14th September can be authorised by a superintendent.
* The male can be detained only until 14.45 hours on 14th September unless his detention is lawfully extended. An extension to detain until 02.45 hours on the 15th September can be authorised by a superintendent.
* The male can be detained only until 15.05 hours on 14 September unless his detention is lawfully extended. An extension to detain until 03.05 hours on the 15th September can be authorised by a superintendent.
* The male can be detained only until 02.45 on 14th September unless his detention is lawfully extended. An extension to detain until 14.45 hours on 14th September can be authorised by a superintendent.
* The male can be detained only until 02.20 hours on 14 September unless his detention is lawfully extended. An extension to detain until 14.20 hours on the 14th September can be authorised by a superintendent.

A
  • The male can be detained only until 02.45 on 14th September unless his detention is lawfully extended. An extension to detain until 14.45 hours on 14th September can be authorised by a superintendent.

Correct. This answer best reflects the position in relation the 24 hour detention time limit and the extension of it in certain circumstances. The relevant time for working out the detention time limits under s.41 PACE and periods of extension under s.42 PACE, is the time of arrival at the police station. The relevant time limit is 24 hours from the time of arrival. This can be extended up to 36 hours in certain circumstances. Whilst other answer options might sound plausible, they are each incorrect. It is incorrect to state that the male can be detained only until 03.05 hours on the 14th September unless his detention is lawfully extended. This is because 03.05 hours was the time that detention was authorised and not the time he arrived at the police station. It is also incorrect to calculate the extension based on the time of his detention. It is incorrect to state that the male can be detained only until 02.20 hours on the 14th September unless his detention is lawfully extended. This is because 02.20 hours was the time of his arrest and not the time he arrived at the police station. There was no prolonged delay between his arrest and arriving at the police station. It is also incorrect to calculate the extension based on the time of arrest. It is incorrect to state that the male can be detained until 14.45 hours on the 14th September unless his detention is lawfully extended. This is because 14.45 hours is 36 hours after the time of arrival at the police station and he can only be detained for 24 hours after his time of arrival unless his detention is lawfully extended. It is incorrect to state that the male can be detained until 15.05 hours on the 14th September unless his detention is lawfully extended. This is incorrect for two reasons. First, the time limit has been calculated from the time of detention rather than the time of arrival. Secondly, the time limit prior to any extensions is 24 hours from the time of arrival. Please review your materials on Police Powers and detention time limits including when those time limits can be extended. You should refer to s.41 and s.42 PACE 1984.

How well did you know this?
1
Not at all
2
3
4
5
Perfectly
6
Q

You act for a defendant due to stand trial for a charge of common assault on their neighbour. Two weeks prior to the trial the defendant attends your office to provide you with full instructions in preparation for the trial during which they admit they assaulted the victim ‘because they deserved it’. The defendant instructs that they wish to maintain the not guilty plea and tell the court that he acted in self-defence.

Can you continue to act for the defendant?
Select one alternative:
* No. You cannot continue to act now that you know that he is guilty.
* Yes. You can continue to act but the defendant must not assert his innocence during the trial.
* Yes. You can continue to act for him even if he raises self-defence at his trial.
* No. You cannot continue to act unless the defendant is willing to tell the court the truth.
* Yes. You can continue to act but in accordance with the overriding objective you will need to notify the court of his recent instructions.

A
  • Yes. You can continue to act but the defendant must not assert his innocence during the trial.

Correct. You could continue to act provided you do not mislead the court. A defendant is entitled to put the prosecution to proof and this is a fundamental right under the adversarial system. They cannot at any stage in the course of the proceedings, assert their innocence nor can you do so on their behalf as to do this would be misleading the court CCS 1. Although there is a duty to assist the court this does not override confidentiality. Without the defendant’s consent you cannot notify the court of their recent instructions. Having admitted the offence to you, you cannot continue to act if they raise self-defence at trial.

How well did you know this?
1
Not at all
2
3
4
5
Perfectly
7
Q

Question
Your client has been charged with an offence of grievous bodily harm contrary to s.20 Offences Against the Person Act 1861 and is due to appear in the magistrates’ court. Your client says he is not guilty as he acted in self-defence. He would like to know what will happen when he makes his first appearance in the magistrates’ court for this offence.
Which of the following best summarises the advice you would give to your client?
Select one alternative:
* Your client will enter his plea of not guilty, the court will then go through the allocation procedure and determine that the trial must take place in the magistrates’ court, because of the nature of the offence.
* Your client will enter his plea of not guilty, the court will then go through the allocation procedure and determine that the trial must take place in the crown court, because of the nature of the offence.
* Your client will enter his plea of not guilty at the crown court as his matter cannot be dealt with in the magistrates’ court given the nature of the offence.
* Your client will indicate his plea of not guilty, the court will then go through the allocation procedure and determine whether the trial can take place on the same day or at a future date.
* Your client will enter his plea of not guilty, the court will then go through the allocation procedure to determine the most suitable venue for trial.

A
  • Your client will enter his plea of not guilty, the court will then go through the allocation procedure to determine the most suitable venue for trial.

Correct. As your client is raising self-defence you should advise him to enter a not guitly plea in the Magistrates’ court. Section 20 Offences Against the Person Act 1861 is an either way offence so the court will need to go through the allocation procedure and determine the most suitable venue. Whilst the other answers sound plausible, they are each incorrect. It is incorrect to state that the court will go through the allocation procedure and then determine that the trial must take place in the magistrates’ court because of the nature of the offence. The nature of the offence does not mean that the court must allocate the trial to the magistrates’ court. There are a number of factors that will be considered during the allocation process in order to determine the most suitable venue. It is incorrect to state that the court will go through the allocation procedure and then determine that the trial must take place in the crown court because of the nature of the offence. The nature of the offence does not mean that the court must allocate the trial to the crown court. There are a number of factors that will be considered during the allocation process in order to determine the most suitable venue. It is incorrect to state that the court will go through the allocation procedure and determine whether the trial can take place on the same day or a future date. The trial will not be able to take place on the same day as the parties will not be prepared to proceed with a trial of this nature and the witnesses are unlikely to be in attendance. It is incorrect to state that the client must enter his plea at the crown court as the magistrates’ court are unable to deal with this offence. Section 20 is an either way offence not an indictable only offence; this means it can either be heard at the magistrates’ court or the crown court. Your client will be expected to enter a plea and then the court will go through the allocation process to determine the most suitable venue for the trial. Please review your materials on plea before venue hearings and the classification of offences. You should review the procedure that takes place at a plea before venue hearing and revisit your materials on the allocation process.

How well did you know this?
1
Not at all
2
3
4
5
Perfectly
8
Q

Question
A defendant is appearing before the magistrates’ court having been arrested for having failed to surrender to the court three weeks previously. The defendant says to the court that at the time she was due in court she was actually in hospital.
Which of the following best reflects whether the defendant has committed an offence of failing to surrender to custody?
Select one alternative:
* The defendant will have a defence to the offence of failing to surrender to custody; it is for the prosecution to prove that the defendant was not in the hospital at the time of the hearing.
* The defendant will have a defence to the offence of failing to surrender to custody; there is no need for the defendant to prove anything.
* The defendant will have a defence to the offence of failing to surrender to custody, if she can prove that she had a reasonable cause by being in hospital.
* The defendant will have a defence to the offence of failing to surrender to custody; it is for the prosecution to prove that the defendant did not have a reasonable cause for failing to surrender.
* The defendant will have a defence to the offence of failing to surrender to cusotdy; it is for the defendant’s solicitor to make represenations to persuade the court that the defendant had a reasonable cause for failing to surrender.

A
  • The defendant will have a defence to the offence of failing to surrender to custody, if she can prove that she had a reasonable cause by being in hospital.

Correct. The offence is committed if the defendant fails to surrender to custody without reasonable cause. The burden is on the defendant to prove that she had a reasonable cause for failing to surrender on the balance of probabilities. If the defendant can prove this on the balance of probabilities, she will not be convicted of the offence. If the defendant is unable to prove that she had reasonable cause for failing to surrender, she will be convicted and sentenced for the offence and it can be considered by the court in any future bail applications. Whilst the other answers may sound plausible, they are each incorrect. It is incorrect to state that it is for the prosecution to prove that the defendant was not in the hospital at the time of the hearing. The defendant bears the burden of proving that they were in the hospital and therefore had a reasonable cause for failing to surrender. It is incorrect to state that it is for the prosecution to prove that the defendant did not have a reasonable cause for failing to surrender. The defendant bears the burden of proving that they were in the hospital and therefore had a reasonable cause for failing to surrender. It is incorrect to state that there is no need for the defendant to prove anything. As the defendant is seeking to rely on having a reasonable cause for failing to surrender, they must prove what that reasonable cause was. Whilst the defendant’s solicitor is likely to make representations about the defendant’s reasonable cause for failing to surrender, the court are likely to request proof of the hospital attendance. Therefore, this is not the single best answer. Please review your materials on bail, the offence of failure to surrender and who bears the burden of proving reasonable cause. Please also review your materials on the relationship between a conviction for failure to surrender and any subsequent bail applications.

How well did you know this?
1
Not at all
2
3
4
5
Perfectly
9
Q

Question
The defendant has been arrested on suspicion of Assault Occasioning Actual Bodily Harm contrary to s. 47 Offences against the Person Act 1861. It is alleged that he punched the victim in the face and head during a fight. He has some blood on his clothing and scratches on his knuckles. In consultation with his solicitor he denies being a perpetrator and says that he, in fact,was a victim in the incident, and indicates that he will plead not guilty at trial. He says that that blood on his clothing is his own as he was punched on the nose during the fight.
Which of the following statements best describe the possible consequences of the defendant remaining silent in interview?
Select one alternative:
* If the defendant exercises his right of silence in interview, he runs the risk of an adverse inference being drawn at trial under s.34 CJPOA 1994, only if he remains silent at trial.
* If the defendant exercises his right of silence in interview, he runs the risk of an adverse inference being drawn under s.34 CJPOA, unless he states that he could not have known the answers to the questions during the interview.
* If the defendant exercises his right of silence in interview, he runs the risk of an adverse inference being drawn under s.34 CJPOA, unless he can show that the police failed to provide full disclosure to his solicitor prior to the interview.
* If the defendant exercises his right of silence in interview, he runs the risk of an adverse inference being drawn at trial under s.34 CJPOA 1994, if he goes on to raise self-defence.
* If the defendant exercises his right of silence in interview, he runs the risk of an adverse inference being drawn under s.34 CJPOA, unless he states that he relied on legal advice to remain silent.

A
  • If the defendant exercises his right of silence in interview, he runs the risk of an adverse inference being drawn at trial under s.34 CJPOA 1994, if he goes on to raise self-defence.

Correct. The answer states that there is a possibility of an adverse inference being drawn at trial if he remains silent in his police interview but later goes on to raise self-defence, setting out that he was the victim of the assault and the blood found on his clothes was his own. The adverse inference will be drawn under s.34 Criminal Justice and Public Order Act 1994 (‘CJPOA’). Whilst the other answer options sound plausible, they are each incorrect. It is incorrect to state that the defendant only runs the risk of an adverse inference being drawn under s.34 CJPOA if he continues to remain silent at trial. A S.34 CJPOA inference will only be drawn where the defendant is silent in interview but later puts forward an account during the trial. If the defendant remains silent at trial an adverse inference may be drawn under s.35 CJPOA. It is incorrect to state that the defendant runs the risk of an adverse inference being drawn under s.34 CJPOA unless he can show that he did not know the answers to the questions during the interview. Whilst it is correct that the defendant can only be expected to answer questions that he could have reasonably known at the time of the questions being asked, that does not apply on these facts. The defendant would have known, at the time of the interview, that he was the victim of the fight and the blood on his clothes belonged to him. It is incorrect to state that the defendant runs the risk of an adverse inference being drawn under s.34 CJPOA unless he states that he relied on legal advice to remain silent. Reliance on legal advice is not enough on its own to prevent an adverse inference being drawn. It is incorrect to state that the defendant runs the risk of an adverse inference being drawn under s.34 CJPOA unless he can show that the police failed to provide full disclosure to the solicitor. This is incorrect because the police do not have an obligation to provide full disclosure to the solicitor and this is not a reason to avoid an adverse inference being drawn. Please review your materials on adverse inferences, particularly s.34 CJPOA 1994. You should also review your materials on s.35, s.36, s.37 CJPOA.

How well did you know this?
1
Not at all
2
3
4
5
Perfectly
10
Q

The defendant has been arrested in relation to an offence of assault occasioning actual bodily harm contrary to s.47 Offences Against the Person Act 1861. He was arrested after police were called to an incident late at night at a local park whereby witnesses had seen a two youths engaged in a fight. The defendant was arrested on a street corner close to the incident. The defendant tells you he was there because he had been to visit a girl who isn’t his girlfriend. He doesn’t want to admit this to the police as he is worried that his girlfriend will find out that he has been cheating on her.
Which of the following best describes the advice you would give the defendant in relation to whether he should account for his presence at the scene at the time of the alleged assault?
Select one alternative:
* You should advise the defendant that he has a right to silence. Based on the account that he has provided to you, this does not provide him with a credible reason for his presence. He should not answer police questions.
* You should advise the defendant that he has a right to silence. Based on the account that he has provided, he does not need to answer questions because he has exercised his right to legal advice.
* You should advise the defendant that he has a right to silence. Based on the account that he has provided however, he should answer police questions but only relating to why he was present at the scene.
* You should advise the defendant that he has a right to silence. Based on the account that he has provided to you, it will portray him as dishonest, so he should not answer police questions.
* You should advise the defendant that he has a right to silence. Based on the account that he has provided to you, however, he should answer police questions to account for his presence.

A
  • You should advise the defendant that he has a right to silence. Based on the account that he has provided to you, however, he should answer police questions to account for his presence.

Correct. This answer correctly states that the defendant has a right to silence. A failure to account for his presence at the scene could lead to an adverse inference being drawn under s.37 Criminal Justice and Public Order Act 1994 (‘CJPOA’). If you are confident that his reason for being at the scene is a legitimate one, then you should advise your client to answer questions so he provide the police with his reason for being present at the scene and avoid an adverse inference being drawn against him. Whilst the other answers are plausible; they are each incorrect. It is incorrect to state that the defendant should remain silent as he has not provided a credible reason for his presence at the scene. You have failed to mention that this would lead to an adverse inference being drawn against him under s.37 Criminal Justice and Public Order Act (‘CJPOA’). It is incorrect to state that the defendant should remain silent as he is likely to come across as dishonest if he provides his reason for being present at the scene. You have failed to mention that this would lead to an adverse inference being drawn against him under s.37 Criminal Justice and Public Order Act (‘CJPOA’). It is incorrect to advise the client to only answer the questions relating to his presence at the scene. You must advise of the fact that an adverse inference could be drawn, under s.34 Criminal Justice and Public Order Act (‘CJPOA’), if this were the case as the court would want to know why he answered some questions in interview but not others. It is incorrect to state that the defendant does not need to answer questions as they have exercised their right to legal advice. An adverse inference can still be drawn under s.37 Criminal Justice and Public Order Act (‘CJPOA’) whether the client obtained legal advice or not. Please review your materials on advising a suspect at the police station for interview and adverse inferences.

How well did you know this?
1
Not at all
2
3
4
5
Perfectly
11
Q

Question
Two women, both aged 18, are in police custody. they were arrested on suspicion of criminal damage, having thrown stones at the windows of a hostel causing a number of them to break. No one else is suspected of involvement in the incident, and nothing appears to have been stolen. Reliable witness evidence clearly identifies both women, and one witness also filmed them on his phone. The police have taken witness statements from all of the witnesses and the footage from the phone is with the police. Both women have been told that a decision has been taken to delay their right to legal advice.
Which of these answers best sets out whether there are reasonable grounds to delay this right?
Select one alternative:
* If all the relevant criteria are met, then the police have a right to delay the women contacting their solicitor.
* There may be reasonable grounds for the delay because there is a risk that allowing access to legal advice will lead to the solicitor inadvertently or otherwise passing on information which might cause interference with evidence.
* There may be reasonable grounds for the delay, given that both women have been identified by reliable evidence.
* There are no obvious reasonable grounds to delay access to legal advice. The decision may be unlawful on that basis.
* There may be reasonable grounds for believing that allowing access to legal advice might lead to the solicitor inadvertently or otherwise passing on information alerting other persons suspected of involvement in the offence.

A
  • There are no obvious reasonable grounds to delay access to legal advice. The decision may be unlawful on that basis.

Correct. For a delay to be lawful, the decision making officer must have reasonable grounds to believe that allowing access to legal advice will lead to the solicitor inadvertently or otherwise passing on information likely to lead to one of the consequences in s. 58(8) or 58(8A) PACE 1984. No such risk appears to apply here. Please review your materials on the rights of a suspect at the police station, in particular the right to access legal advice s.58 PACE 1984.

How well did you know this?
1
Not at all
2
3
4
5
Perfectly
12
Q

Question
Your client has been arrested for an offence of robbery contrary to s.8(1) Theft Act 1968. It is alleged that your client threatened to punch the victim in the face unless he handed over his wallet. The victim states that he handed over his wallet containing £50 cash, a debit card, credit card and various store loyalty cards.
Your client was arrested two hours after the incident in the street adjacent to where the robbery took place after she matched the description provided by the victim. Upon searching your client, a credit card in the name of the victim was found in her front jacket pocket. Your client tells you she found the credit card and was about to hand it to the police station. Your client is to be interviewed under caution.
Which of the following best summarises the advice you would give to your client?
Select one alternative:
* Under s.37 Criminal Justice and Public Order Act 1994 (‘CJPOA’) a court can draw an adverse inference if your client fails to account for the credit card in her possession provided a special warning is given.
* Under s.36 Criminal Justice and Public Ordder Act 1994 (‘CJPOA’) a court can draw an adverse inference if your client fails to account for the credit card in her possession. No special warning is required in these circumstances as she has a solicitor to provide legal advice.
* Under s.37 Criminal Justice and Public Order Act 1994 (‘CJPOA’) a court can draw an adverse inference if your client fails to account for the credit card in her possession. No special warning is required in these circumstances as she has a solicitor to provide legal advice.
* Under s.36 Criminal Justice and Public Order Act 1994 (‘CJPOA’) a court can draw an adverse inference if your client fails to account for the credit card in her possession provided a special warning is given.
* Under s.35 Criminal Justice and Public Order Act 1994 (‘CJPOA’) a court can draw an adverse inference if your client fails to account for the credit card in her possession provided a special warning is given.

A
  • Under s.36 Criminal Justice and Public Order Act 1994 (‘CJPOA’) a court can draw an adverse inference if your client fails to account for the credit card in her possession provided a special warning is given.

Correct. This answer correctly reflects the best advice to provide to your client. As she has been found in possession of the credit card, an adverse inference can be drawn under s.36 CJPOA if she fails to account for having the credit card in her possession. A special warning must be given. Whilst the other answers are plausible, they are each incorrect. It is incorrect to state that the court may draw an adverse inference under s.37 CJPOA if your client fails to account for the possession of the credit card provided a special warning is given. Section 37 CJPOA relates to a suspects failure to account for their presence at the scene not possession of an object. It is correct to state that, under s.36 CJPOA, the court can draw an adverse inference for the clients failure to account for possession of the credit card. It is incorrect to state that no special warning is required where the suspect has legal advice. It is incorrect to state that, under s.37 CJPOA, the court can draw an adverse inference for the client’s failure to account for possession of the credit card. Section 37 CJPOA relates to a suspect’s failure to account for their presence at the scene not possession of an object. It is also incorrect to state that no special warning is required where the suspect has legal advice. It is incorrect to state that, under s.35 CJPOA, the court can draw an adverse inference for the clients failure to account for possession of the credit card. Section 35 CJPOA relates to silence at trial. Please review your materials on adverse inferences and the relevant sections of the CJPOA.

How well did you know this?
1
Not at all
2
3
4
5
Perfectly
13
Q

Question
Your client is on trial for theft. The only evidence against him is that of a witness (who does not know your client) who claims that she saw him take a purse from the victim’s handbag. During cross-examination, the witness concedes that the incident took place after dark in a busy street about 40 yards from where she was standing. She also concedes that her view was partially obstructed by passers-by and that she cannot now be sure that it was your client she saw taking the purse.
Which of the following is the most appropriate course of action for the judge to take in this situation?
Select one alternative:
* The Judge should withdraw the case from the jury unless they feel that defendant would receive a fair trial if they gave a Turnbull warning.
* The Judge should allow the case to go to the jury but give a Turnbull warning.
* The Judge should allow the case to go to the jury but exclude the evidence of the identification.
* The Judge should withdraw the case from the jury and direct a conviction.
* The Judge should withdraw the case from the jury and direct an acquittal.

A

The Judge should withdraw the case from the jury and direct an acquittal.

Correct. Although the judge could allow the case to go to the jury, the witness has conceded that she cannot be sure of the identification, and as such a jury properly directed could not safely convict. The most appropriate course of action is therefore to withdraw the case from the jury under the R v Galbraith principles and direct an acquittal. Whilst the other answers might sound plausible, they are each incorrect. It is incorrect to state that the Judge should allow the case to go to the jury but give a Turnbull warning as the witness has conceded that she cannot be sure of the identification during her evidence, as such a jury properly directed should not convict. The case should be withdrawn from the jury and they should be directed to acquit. It is incorrect to state that the Judge should allow the case to go to the jury but exclude the evidence of identification. There are a number of reasons why this is incorrect. We are told that the identification evidence is the only evidence against the defendant so there would be nothing left for the jury to consider. The jury has also heard the identification evidence and the witness conceded that she cannot be sure of the identification, as such a jury properly directed could not safely convict. The case should be withdrawn from the jury and they should be directed to acquit. It is incorrect to state that the Judge should withdraw the case from the jury and direct the jury to convict. The witness has conceded that she cannot be sure of the identification therefore there is no basis on which the jury could properly convict the defendant. The Judge should withdraw the case from the jury and they should be directed to acquit. It is incorrect to state that the Judge should withdraw the case from the jury unless they feel that the defendant would receive a fair trial if they gave a Turnbull warning. The witness has conceded that she cannot be sure of the identification, as such a jury properly directed could not convict. The case should be withdrawn from the jury and they should be directed to acquit. Please review your materials on Identification evidence, the Turnbull warning and the test where there is no case to answer in R v Galbraith.

How well did you know this?
1
Not at all
2
3
4
5
Perfectly
14
Q

The defendant has been arrested for an offence of theft contrary to s. 1 of the Theft Act 1968 at 09.30 hours on 12 December. He arrived at the station at 09.55 hours and his detention was authorised by the Custody Sergeant at 10.10 hours.
Which of the following statements best describes the police’s obligation in respect of reviews of the defendant’s detention?
Select one alternative:
* The defendant’s detention must be reviewed by the Review Officer no later than 18.30 hours on 12 December.
* The defendant’s detention must be reviewed by the Review Officer no later than 15.30 hours on 12 December.
* The defendant’s detention must be reviewed by the Review Officer no later than 19.10 hours on 12 December
* The defendant’s detention must be reviewed by the Review Officer no later than 16.10 hours on 12 December.
* The defendant’s detention must be reviewed by the Review Officer no later than 15.55 hours on 12 December.

A
  • The defendant’s detention must be reviewed by the Review Officer no later than 16.10 hours on 12 December.

Correct. This answer correctly states the time period within which the first review must be carried out in accordance with s.40 Police Criminal Evidence Act 1984 (‘PACE’). It correctly calculates that time period from the time detention was authorised. Whilst the other answer options sound plausible, they are each incorrect. It is incorrect to state that the defendant’s detention must be reviewed no later than 15.30hours on the 12th December as this calculates the time for review from the time of arrest not the time detention was authorised. It is incorrect to state that the defendant’s detention must be reviewed no later than 18.30hours on the 12th December as this uses the wrong time period to calculate review. It also calculates the time for review from the time of arrest rather than the time the detention was authorised. It is incorrect to state that the defendant’s detention must be reviewed no later than 15.55hours on the 12th December as this calculates the time for review from the time of arrival at the police station not the time detention was authorised. It is incorrect to state that the defendant’s detention must be reviewed no later than 19.10hours on the 12th December as this uses the wrong time period to calculate review. Please review your materials on detention review periods and s.40 PACE. You should review the time period within which the first review should take place and when that time is calculated from as well as the time period within which any subsequent reviews ought to take place.

How well did you know this?
1
Not at all
2
3
4
5
Perfectly
15
Q

Question
Your client is under arrest for s.18 GBH. Your client matches the description given by the victim of the assault. Your client instructs you that they refuse to participate in a video identification procedure.
Which of the following best reflects what action the police can do?
Select one alternative:
* The police can proceed with a video identification procedure without the client’s consent.
* The police can proceed with a group identification but cannot proceed with a video identification procedure without the clients consent.
* The police can proceed by arranging for a confrontation between the witness(es) and your client but cannot prcoeed with any other form of identification procedure without the clients consent.
* The police can proceed with an identification parade but cannot proceed with a video identification procedure without the clients consent.
* The police cannot proceed with any identification procedure without the clients consent.

A
  • The police can proceed with a video identification procedure without the client’s consent.

Correct. This answer is correct because a video identification procedure can be arranged without the client’s consent. The police will seek the client’s consent but if this cannot be obtained then they will arrange for an existing image of your client to be used for the video identification procedure. In such circumstances, the police are not obliged to give the suspect an opportunity to view the images used in that video identification procedure. On the facts of this case, we are told that the victim of the assault has provided a description of the person who carried out the offence and that the suspect matches that description. This is, therefore, a case where a video identification procedure would serve a useful purpose. Whilst the other answers are plausible, they are all incorrect. It is incorrect to state that the police cannot proceed with any identification procedure without the client’s consent. As mentioned above, the police do not require the client’s consent in order to proceed. It is correct to state that the police can proceed with an identification parade. It is incorrect to state that they are unable to proceed with a video identification procedure due to the lack of consent from the suspect. It is correct to state that the police can proceed with a group identification. It is incorrect to state that the police are unable to proceed with a video identification procedure due to the lack of consent from the suspect. Whilst the police can proceed by arranging a confrontation between the witness and the suspect, this is unlikely to be appropriate in this case as the witness is the victim of the offence. It is also incorrect to state that the police cannot proceed with any other form of identification procedure without the client’s consent. Please review your materials on Identification procedures including what happens in circumstances where a suspect refuses to take part in a video identification procedure and COP D 3.21.

How well did you know this?
1
Not at all
2
3
4
5
Perfectly
16
Q

Question
The defendant is on trial in the Crown Court for assault occasioning actual bodily harm against her boyfriend after an altercation at a local nightspot, called the Bar. Her boyfriend has refused to co-operate with police and the only witness contends that the defendant was at the Bar at the time of the assault and that he saw her hit a male, who is now accepted to be her boyfriend. The witness points the defendant out to a police officer in the street the next morning. The defendant’s defence is that she was at home all evening with her boyfriend and their dog.

The trial is just about to begin. The officer in the case has admitted that the witness was not asked to take part in a formal Identification procedure as required by Code D of PACE.

What is the best approach for the defence counsel to take in this situation?
Select one alternative:
* Defence counsel should make the point about the lack of identification procedure in the closing speech.
* Defence counsel should make an application for the case to be dismissed as an abuse of process.
* Defence counsel should apply to the judge to exclude the witness identification evidence under s.78 PACE 1984.
* Defence counsel should cross examine the witness on the fact that he had not taken part in a formal ID procedure.
* Defence counsel should make a submission of no case to answer on the basis that the jury cannot safely convict as the witness identification has not been properly tested with a formal ID procedure.

A
  • Defence counsel should apply to the judge to exclude the witness identification evidence under s.78 PACE 1984.
How well did you know this?
1
Not at all
2
3
4
5
Perfectly
17
Q

Question

You are contacted by the DSCC in relation to two suspects who have been arrested in relation to a suspected complex fraud at their place of work. The suspects are co-workers.
Which of the following best explains whether you and/ or your firm can act for both suspects? Your firm has a criminal contract with the Legal Aid Agency.
Select one alternative:
* You and your firm can act for both suspects provided the DSCC is satisfied there is no conflict of interests between the suspects.
* Your firm should act for both suspects if they have substantial common interests but each suspect will need separate legal advisers at the police station.
* Your firm can act for both suspects provided there are information barriers in place to protect confidential information
* You and your firm should act for both suspects unless there is a conflict of interest or risk of one.
* A firm cannot act for two suspects accused of the same offence.

A
  • You and your firm should act for both suspects unless there is a conflict of interest or risk of one.

Correct. In publicly funded cases, regulations require that one litigator be appointed to act for all co-defendants in a legal aid case unless there is, or is likely to be, a conflict of interest. The SRA Code of Practice paragraph 6.2 sets out the prohibition on acting in conflict situations and requires that you do not act if you have a conflict of interest or a significant risk of a conflict of such a conflict. The exceptions in paragraphs 6.2(a) and (b) are not relevant in this context.

18
Q

Question
Your client’s trial for robbery is due to begin at the Crown court. The case against her rests substantially, although not wholly, on a disputed identification.
Which of the following statements best summarises the Judge’s duties with regard to the identification evidence in this case?
Select one alternative:
* The Judge will not give the jury a special warning as the case against her is not wholly based upon a disputed identification.
* The Judge will withdraw the evidence from the jury and direct them to acquit if they consider that the identification evidence is weak.
* The Judge will direct the jury to consider whether the defendant has proved that the identification evidence is weak.
* The Judge will give the jury a special warning in relation to the identification evidence as the case against her rests substantially upon a disputed identification.
* The Judge will not give the jury a special warning if they consider that the identification evidence is strong.

A
  • The Judge will give the jury a special warning in relation to the identification evidence as the case against her rests substantially upon a disputed identification.

Correct. As the case rests substantially on disputed identification evidence the Judge should give the jury a special warning in accordance with R v Turnbull. Whilst the other answers sound plausible, they are each incorrect. It is incorrect to state that the Judge will not give the jury a special warning as the case against her is not wholly based upon disputed identification evidence. The guidelines in R v Turnbull state that a special warning should be given where the case rests substantially on disputed identification evidence, not wholly on it. It is incorrect to state that the Judge should withdraw the evidence from the jury and direct them to acquit if they consider the identification evidence to be weak. If the Judge was of the view that the identification evidence is weak but supported by other evidence, they would not withdraw it. The Judge should give the jury the special warning in accordance with the case of R v Turnbull. If the Judge considered the identification evidence to be weak and the case was unsupported by other evidence, then they would likely withdraw it from the jury and direct them to acquit. It is incorrect to state that the Judge will direct the jury to consider whether the defendant has proved that the identification evidence is weak. Although the defendant disputes the identification evidence, the defence does not have a burden of proof. It is for the prosecution to prove the case against the defendant beyond reasonable doubt. As the identification evidence is disputed, the Judge should give the jury a special warning in accordance with R v Turnbull. It is incorrect to state that the Judge will not give a special warning if they consider the identification evidence to be strong. As the identification evidence is disputed, the Judge should give a special warning in accordance with R v Turnbull. Please review your materials on identification evidence and the Turnbull guidelines.

19
Q

Question
Your client is 19 years of age. It is alleged that he punched his best friend, following a disagreement in the street. When his mother asked him whether he punched his friend, your client responded by saying ‘maybe.’ When the police arrive to arrest your client, his mother repeats what her son had said to her. The investigating officer believes that this is a confession.

Does your client’s response to his mother amount to a confession?
Select one alternative:
* Your client’s response does not amount to a confession because he has not written it down and signed it
* Your client’s response does amount to a confession because it is partly adverse to him.
* Your client’s response does amount to a confession because it is an unequivocal acceptance of guilt.
* Your client’s response does not amount to a confession because it is equivocal.
* Your client’s response does not amount to a confession because it has not been made to a person in authority.

A
  • Your client’s response does amount to a confession because it is partly adverse to him.

Correct. This would amount to a confession as his mother asks him if he committed the offence and responded ‘maybe.’ That statement is partly adverse to him and therefore falls within the definition of a confession under s. 82 Police and Criminal Evidence Act 1984 (PACE). Whilst the other answer options sound plausible, they are each incorrect. It is incorrect to state that your client’s response does not amount to a confession because it is equivocal. Whilst the response ‘maybe’ is equivocal, it is partly adverse to your client and therefore falls within the definition of a confession under s.82 PACE. It is incorrect to state that your client’s response does not amount to a confession because it has not been made to a person in authority. A confession does not have to be made to a person in authority. Refer to the full definition under s.82 PACE. It is incorrect to state that your client’s response amounts to a confession because it is an unequivocal acceptance of guilt. The word ‘maybe’ is not an unequivocal statement or an acceptance of guilt. Your client’s response is a confession because the statement made is partly adverse to him and falls within the definition of a confession under s.82 PACE. It is incorrect to state that your client’s response does not amount to a confession because he has not written it down or signed it. There is no requirement for your client to have written the statement down and signed it in order for it to be considered a confession. Please review your materials on confessions and the definition of a confession under s.82 PACE.

20
Q

Question
A man confesses to an offence of causing grievous bodily harm with intent contrary to s.18 Offences Against the Person Act 1861. He subsequently tells his solicitor that he only made the confession because shortly beforehand, during his arrest, a police officer grabbed him by the throat and warned him that he would get ‘worse treatment’ unless he ‘co-operated’. At that stage the man was already handcuffed.
Which of the following best describes the position regarding the admissibility of the confession evidence?
Select one alternative:
* The officer having grabbed the man by the throat will not constitute oppression, as it does not amount to torture. The confession will be used as evidence against him.
* The officer having grabbed the man by the throat may constitute oppression. If the defence can prove this, and prove that the oppression caused the confession, then the court must exclude the confession from the evidence.
* The officer having grabbed the man by the throat may constitute oppression. If so, and if that caused the confession, then the court must exclude the confession from the evidence.
* The officer having grabbed the man by the throat may constitute oppression. If so, and if that caused the confession, then the court has a discretionary power to exclude the confession from the evidence.
* The officer having grabbed the man by the throat may constitute oppression. If the court is satisfied on the balance of probabilities that oppression caused the confession, then the court must exclude the confession from the evidence.

A
  • The officer having grabbed the man by the throat may constitute oppression. If so, and if that caused the confession, then the court must exclude the confession from the evidence.

Correct. The officer grabbing the man by the throat and threatening him whilst he was in handcuffs may amount to oppression under s.76(8) Police and Criminal Evidence Act 1984 (PACE). If it can be shown that this is oppression and this caused the man to confess then the court must exclude the confession under s.76(2)(a) PACE 1984. Whilst the other answer options sound plausible, they are each incorrect. It is incorrect to state that the officer grabbing the man by the throat will not constitute oppression, as it does not amount to torture. Whilst the definition of oppression does refer to torture, it is not limited to torture. It may be that the officer grabbing the man by the throat amounts to oppression if it falls within the definition under s.76(8) PACE. It is incorrect to state that if the officer grabbing the man by the throat amounts to oppression and the confession was caused by that oppression, that the court have a discretionary power to exclude the confession evidence. If the officer’s actions are found to amount to oppression and this caused the confession then the court must exclude the confession evidence. It is incorrect to state that the defence would have to prove that the officer grabbing the man by the throat amounts to oppression and that this caused the man to confess. If the defence raises the issue of oppression, it is for the prosecution to prove that the confession was not obtained using oppression. The standard of proof on the prosecution is beyond reasonable doubt. It is incorrect to state that if the court is satisfied on the balance of probabilities that the confession was obtained by oppression then the court must exclude the evidence. It is for the prosecution to prove beyond reasonable doubt that the confession was not obtained using oppression. If they cannot do this then the court would find that the confession was obtained using oppression and must exclude it. Please review your materials on confession evidence and the courts power to exclude any confession evidence that has been obtained using oppression. You should review the relevant sections of PACE.

21
Q

Your client is a 16 year old youth. Their co-defendant is a 45 year old male. They are jointly charged with a wounding contrary to s.20 OAPA. The adult co-defendant pleads guilty and is committed for sentence to the Crown Court.
Which of the following best represents where the youth’s case will be heard?
Select one alternative:

Your client will be remitted to the Youth Court because the co-defendant is not having a trial.

Your client will be sent to the Crown Court with the co-defendant because they are charged with a grave crime.

Your client will not be sent to the Crown Court with the co-defendant because of the age difference between them.

Your client will be sent to the Crown Court with the co-defendant in the interests of justice.

Your client will be remitted to the Youth Court for trial in the interests of justice.

A

This is correct. The only reason that the youth is in the magistrates’ court is because they are jointly charged with the adult. Without the adult who has been committed to the Crown Court they cannot stay there. The youth can only be sent to either the Crown Court or the youth court. The youth’s circumstances do not satisfy any of the criteria to be sent to the Crown Court. It is not homicide, firearms etc. Section 20 OAPA is not a grave crime as it is not punishable with 14 years as an adult. As the adult has pleaded guilty there will be no joint trial. The interests of justice test would only be applied where both the adult and youth plead not guilty, and the adult is sent to the Crown Court for trial.

22
Q

Your client is due to appear on trial in the Crown Court facing one count of robbery. It is alleged that your client is a member of a local gang who are known for committing robbery and violent offences. On this occasion, they are alleged to have entered the local post office and threatened the shopkeeper unless she handed over the contents of the cash register.

The prosecution state that they have a witness who was in the post office at the time and saw the offence take place. The witness claims to know your client and identified him as being one of the people who committed the offence.

You understand from the prosecution that the witness is anxious about the defendant seeing her give evidence and the prosecution are planning to make an application to deal with this.

Which of the following would be the most appropriate application for the prosecution to make for the witness?

Select one alternative:

An application to have the witness evidence read under s.10 Criminal Justice Act 1967.

An application for special measures to allow the witness to give evidence from behind a screen under s.23 Youth Justice and Criminal Evidence Act 1999.

An application to have the witness evidence read under s.9 Criminal Justice Act 1967.

An application for special measures to allow the witness to give evidence by pre-trial visual recorded cross-examination or re-examination under s.28 Youth Justice and Criminal Evidence Act 1999.

An application for special measures to allow the witness to give evidence through an intermediary under s.29 Youth Justice and Criminal Evidence Act 1999.

A

This is correct. An application for special measures for the witness to give evidence from behind screens is the most appropriate application. The evidence would not be agreed or read under either s.10 or s.9 Criminal Justice Act. The other special measures would not be the most appropriate in the circumstances as they are more appropriate for vulnerable witnesses rather than intimidated witnesses.

23
Q

Your client is due to be sentenced in the Crown Court after entering a guilty plea to an offence of burglary (dwelling). He has a previous conviction from 6 years ago when he pleaded guilty to burglary (dwelling) and a previous conviction from last year when he was found guilty after trial for burglary (non-dwelling).

How are the previous convictions likely to affect the court’s decision regarding sentence?

Select one alternative:

The court will have to impose a mandatory minimum sentence of three years as this will be the third burglary conviction for this defendant.

The court will not be able to impose a mandatory minimum sentence as the previous convictions were not committed within the last 5 years.

The court will have to impose a mandatory minimum sentence of seven years as this will be the third burglary conviction for this defendant.

The court will not be able to impose a mandatory minimum sentence as the previous convictions are not all for domestic burglary (dwelling).

The court will not be able to impose a mandatory minimum sentence as the defendant did not plead guilty to all the previous convictions.

A

The court will not be able to impose a mandatory minimum sentence as the previous convictions are not all for domestic burglary (dwelling).

This is correct. The court cannot impose a mandatory minimum sentence where one of the convictions is for burglary (non-dwelling). The mandatory minimum period is not 7 years. The previous convictions do not need to be in the last 5 years for the minimum sentence provisions to apply. The defendant need not have pleaded guilty to all previous relevant offences for the minimum sentence provisions to apply.

24
Q

D is on trial in the magistrates’ court for an offence of shoplifting. The prosecution alleges that D stole a book from the Bookshop. The prosecution has concluded its case but did not adduce any evidence that the book was the Bookshop’s property. You are defending D on instructions that he took the book by mistake having received a call on his mobile phone which distracted him. His brother who rang him is at court ready to give evidence on Ds behalf.
Which ONE of the following would be the MOST APPROPRIATE action for you to take?
Select one alternative:

You should ask the prosecutor in open court whether or not they have any evidence of ownership of the book and if so, whether they intend to use it.

You should not call any defence witnesses and should proceed to your closing speech explaining that D had no intention to permanently deprive the Bookshop of the book and should therefore be acquitted.

You should proceed to call D as your first witness because he is the defendant. D can give evidence that he was not dishonest.

You should not call any defence witnesses and should proceed to your closing speech explaining that D was not dishonest and should therefore be acquitted.

You should make a submission of no case to answer as the prosecution has not proved all the elements of theft

A

You should make a submission of no case to answer as the prosecution has not proved all the elements of theft

This is correct. Where the prosecution case is weak, the defence may make an application to the judge (in the Crown Court) or to the magistrates or District Judge (in the magistrates’ court) to dismiss the case because there is no case to answer. The case of R v Galbraith set out the test for the court to consider when hearing an application for ‘no case to answer’. The submission has two limbs as follows: * that there is no evidence that a crime has been committed by the defendant; or * that there is some evidence before the court, but it is tenuous or inconsistent in nature. Here the argument would be that there is no evidence that the property belonged to another so there is no case to answer. If the submission is successful, the case will come to an end. If unsuccessful then you might call D and his brother to rebut the issue of dishonesty.

25
Q

A man is charged with fraud. The prosecution wants to call his wife as a witness against him. She is not herself under suspicion of having committed any offence. However, she is reluctant to give evidence against her husband.
Which of the following is correct?
Select one alternative:

The wife is a competent witness, but cannot be compelled to give evidence against her husband for this offence. However, if she refuses to give evidence an adverse inference may be drawn against her.

The wife is a competent witness, and can be compelled to give evidence against her husband for this particular offence.

The wife is not a competent witness against her husband.

The wife is a competent witness, but cannot be compelled to give evidence against her husband for this offence.

The wife is a competent witness, but cannot be compelled to give evidence against her husband for this or any offence.

A

The wife is a competent witness, but cannot be compelled to give evidence against her husband for this offence.

The wife is a competent witness for the prosecution (s. 53(1) YJCEA 1999). However, she can only be compelled to give evidence against her husband if any of the exceptions in s. 80(3) PACE 1984 apply (s.80 (2A)(b) PACE). None of the exceptions in s. 80(3) are relevant on the facts and, therefore, the wife cannot be compelled to give evidence against her husband. She can, in some circumstances, be compelled (i.e., where the offence is specified under s. 80 PACE). As the wife is not accused of any offence, no adverse inference may be drawn against her.

26
Q

6
A defendant is charged with burglary (non-dwelling) and has pleaded not guilty at the Plea and Trial Preparation Hearing (PTPH) in the Crown Court. He has two previous convictions, one for a burglary (non-dwelling) from three years ago and a theft from last year. The prosecution have indicated that they are planning to make an application to adduce these previous convictions in evidence at the trial.

Which of the following correctly sets out the procedure the prosecution will need to follow?

Select one alternative:

The prosecution does not need to serve notice to adduce evidence of the defendant’s bad character as they have indicated that they are planning to make an application at trial.

The prosecution must serve notice to adduce evidence of the defendant’s bad character on the court officer and all other parties not more than 28 business days after the defendant pleads not guilty.

The prosecution must serve notice to adduce evidence of the defendant’s bad character on the court officer and all other parties not more than 20 business days after the defendant pleads not guilty.

The prosecution does not need to serve notice to adduce evidence of the defendant’s bad character, the matter will simply be dealt with at trial.

The prosecution must serve notice to adduce evidence of the defendant’s bad character on the court officer and all other parties not more than 10 business days after the defendant pleads not guilty.

A

The prosecution must serve notice to adduce evidence of the defendant’s bad character on the court officer and all other parties not more than 10 business days after the defendant pleads not guilty.

This is the correct answer. Pursuant to CrimPR 21, in a Crown Court case the prosecution must service notice of intention to adduce the defendant’s bad character not more than 10 business days from a not guilty plea. Whilst plausible, the other answers are incorrect. 20 business days is the time limit for service of the notice to adduce the defendant’s bad character in magistrates’ court cases. 28 business days is the incorrect time limit for service of the prosecution notice of intention to adduce the defendant’s bad character. The prosecution does need to serve notice of intention to adduce the defendant’s bad character evidence in relation to the circumstances in this particular scenario.

27
Q

The Prosecution wishes to rely on a defendant’s bad character. D is currently awaiting trial for Common Assault against her neighbour along with her sister who is also a co-defendant. The Prosecution seeks to admit the defendant’s previous conviction for Assault Occasioning Actual Bodily Harm and a misdemeanour of breach of the peace which involved the defendant threatening her previous neighbour.
Which gateway of s101 Criminal Justice Act 2003 will the prosecution use to make the application?
Select one alternative:

Important matter in issue between the defendant and prosecution

Parties agree the evidence as being admissible

It has substantial probative value in relation to an important matter in issue between the defendant and a co-defendant

Propensity to commit offences of the type charged

Important explanatory evidence

A
  • Important matter in issue between the defendant and prosecution

This is correct. It is an important matter in issue between the defendant and prosecution as the prosecution will state it shows propensity to commit the offence of which she is charged. It would be very unusual for a defendant to agree a more serious previous conviction and a similar neighbour dispute to be admitted as bad character as this is likely to be one of the factors towards a conviction. It is not an offence where important information needs to be relied upon to understand the current offence. Section 103(1) is not a gateway to admit bad character. Rather it is to clarify how gateway 101(1)(d) can be satisfied. The gateway under s 101(1)(e), a matter in issue between the defendant and a co-defendant, is a gateway generally invoked by the defence.

28
Q

A man is sentenced in the Crown Court for two unrelated offences contrary to s.20 Offences Against the Person Act 1861 and a separate offence of Assault Occasioning Actual Bodily Harm contrary to s.47 Offences Against the Person Act 1861. He receives a custodial sentence of 12 months for each of the s.20 offences, which are ordered to run consecutively and 6 months for the ABH to run concurrently.

Assuming the man does not commit any offences whilst in prison and therefore only serves half of his sentence in custody, when should the man be released?

Select one alternative:

After serving two and a half years in custody.

After serving one year in custody.

After serving one year and three months in custody.

After serving one and a half years in custody.

After serving two years in custody.

A
  • After serving one year in custody.

This is incorrect. - This question tests your understanding of sentencing and the difference between consecutive and concurrent custodial sentences. Sentences which run consecutively will be served one after the other but sentences served concurrently will be served alongside each other. In this example the two sentences for the s.20 offences will be served one after the other but the sentence for s.47 will be served alongside the sentences for the s.20 offences. This means that the total sentence will equate to two years. As the prisoner will only serve half of their sentence in custody, they will be released from prison after serving one year in custody. Please review your materials on determining sentence, particularly consecutive and concurrent sentences.

29
Q

You are representing a client who is due to make their first appearance before the magistrates’ court before a District Judge.

What is the proper form of address that you should adopt when addressing the court?

Select one alternative:

Your worships

My Lord/Lady

Sir/Ma’am

Your honour

Master

A
  • Sir/Ma’am

This is correct. A District Judge should be referred to as sir/ma’am this is the correct form of address.

30
Q

A solicitor represented a new client in the magistrates’ court at a trial for common assault. The court heard that the defendant was of good character. At the conclusion of the case, the client was convicted. After the trial, the client has told his solicitor that he wishes to appeal against the conviction. He also confides in the solicitor by telling her that the police made an error with his name and date of birth when they charged him. He actually has a number of previous convictions for violent offences.

The client insists that he does not want the court or prosecution to be made aware of his real name and date of birth as he does not want them to find out about his previous convictions.

What should the solicitor do in light of this new information?

Select one alternative:

The solicitor should withdraw from acting for the client and must inform the court of the client’s correct name and date of birth, otherwise she is complicit in misleading the court.

The solicitor can continue to act for the client as she has an overriding to duty to act in the best interests of her client.

The solicitor should withdraw from acting for the client, but she cannot notify the court or prosecution as to the reasons for withdrawing.

The solicitor should withdraw from acting for the client and advise the court that she is doing so as to continue to act would put in her a position whereby she would be misleading the court.
Answered and wrong

The solicitor can continue to act for the client, but she must not make reference to the clients name, date of birth or good character at the appeal hearing.

A
  • The solicitor should withdraw from acting for the client, but she cannot notify the court or prosecution as to the reasons for withdrawing.

This is correct. This question tests the knowledge of professional conduct in criminal litigation and the solicitor’s duty not to mislead the court alongside their duty of confidentiality to the client. The solicitor must cease to act as they could not continue and follow the client’s instructions as it would put the solicitor in breach of their duty to not mislead the court. The solicitor cannot inform the court of the reason for withdrawing otherwise they would breach their duty of confidentiality owed to the client.

31
Q

Your client is 25 years old and twelve months ago he was sentenced to 12 months’ imprisonment suspended for two years with an unpaid work requirement of 180 hours. Last week your client was convicted of a new offence and is now back before the judge for breach of the original suspended sentence. Your client completed 90 of the 180 hours of Unpaid Work.
What must the court do?
Select one alternative:

The court must add another 90 hours of Unpaid Work unless it would be unjust in all the circumstances to do so.

The court must activate half of the custodial sentence unless it would be unjust in all the circumstances to do so.

The court must activate the custodial sentence in full or in part unless there are exceptional circumstances.

The court must activate the custodial sentence unless there are exceptional circumstances.

The court must activate the custodial sentence unless it would be unjust in all the circumstances to do so.

A

The court must activate the custodial sentence unless it would be unjust in all the circumstances to do so.

This is correct. See Criminal Justice Act 2003, Sch.12 (8) (2) (a) which provides for the options available to the court when dealing with a breach of suspended sentence. The court must activate the custodial sentence unless it would be unjust in all the circumstances to do so. Whether there are exceptional circumstances which remove the requirement to activate the original sentence is not the correct test. The court may consider reducing the original sentence to account for partial completion of the community order, but it does not have to do so. If the court does not activate the sentence, it may consider making the community order more onerous.

32
Q

A defendant is charged with burglary and is due to appear before the Crown Court for trial. It is part of the prosecution case that when the police carried out a search at the defendant’s address, a couple who were walking past in the street told the police officers that they lived round the corner and ‘the person who lived at the address was a burglar and wants locking up.’

The police were just about to carry out the search and didn’t take the name and address details from the couple but noted that they said that they lived around the corner. It appears that the couple’s details were never obtained.

The prosecution is seeking to adduce this evidence through the police officer’s witness evidence.

Which of the following best explains the position regarding whether the police officer is able to give evidence of what the couple said to him during the search?

Select one alternative:

The officer cannot give evidence of what the couple said to him as it is hearsay and is therefore inadmissible.

The officer can give evidence of what the couple said to him, this is not hearsay evidence and will be admissible on the basis that it is relevant to a fact in issue.

The officer can give evidence of what the couple said to him, even though it is hearsay evidence, it will be admissible on the basis that it will be in the interests of justice to admit the evidence.

The officer can give evidence of what the couple said to him, even though it is hearsay evidence, it will be admissible on the basis that the witness is unavailable as they cannot be found.

The officer cannot give evidence of what the couple said to him as it is hearsay and is not admissible on the basis that the couple cannot be identified to the court’s satisfaction.

A
  • The officer cannot give evidence of what the couple said to him as it is hearsay and is not admissible on the basis that the couple cannot be identified to the court’s satisfaction.

Correct – this would be hearsay evidence as the prosecution are seeking to prove that the defendant is a burglar and there is a statement of that matter in the communication as the couple have said that a burglar lives at the defendant’s address and they said it intending that the police officers would believe that a burglar lived at that address. Hearsay evidence would be inadmissible unless one of the exceptions to this rule applies. The issue here is that the police have not identified the maker of the statement to the satisfaction of the court and therefore should not be admitted.

33
Q

The defendant was convicted by a jury after a trial for an offence of an occupied, night time, dwelling house burglary. The defendant has one previous conviction for burglary. The defendant was intoxicated at the time.
Which of the following best explains what would be an appropriate consideration in sentencing?
Select one alternative:

The judge will impose a community order as the offence is not so serious.

The judge is entitled to treat the intoxicated state of the defendant as a mitigating factor and reduce a custodial sentence accordingly.

The aggravating factors present will mean the offence passes the custodial threshold.

The judge cannot impose a sentence of more than 6 months as this is one either way offence.

The judge must impose a custodial sentence of three years. It is a mandatory sentence as the defendant has a previous conviction for burglary.

A
  • The aggravating factors present will mean the offence passes the custodial threshold.

This is correct. Burglary is an either-way offence. Here the defendant has been convicted in the Crown Court. Whether a custodial sentence is imposed will depend upon whether the offence passes the custody threshold. A domestic burglary of this nature along with the previous conviction will be so serious that that neither a fine alone nor a community sentence can be justified for the offence. The maximum of 6 months is only relevant in the magistrates’ court where the powers are restricted. The question says jury so this must be in the Crown Court. The mandatory minimum sentence of 3 years is for a third domestic burglary so there would need to be two relevant previous convictions. Commission of the offence whilst intoxicated is an aggravating not a mitigating factor (although genuine efforts to address the addiction can be a mitigating factor).

34
Q

You are representing a 16 year old youth who is appearing in the Youth Court for their first appearance charged with a serious knifepoint robbery of an expensive mobile phone. The victim was highly traumatised by the offence. Your client has no previous convictions. Your client enters a not guilty plea. After hearing representations from the defence and prosecution, the bench come to the view that, in the case of an adult, the matter would be in a sentencing category with a starting point of 4 years and a range of 3-6 years.

Which of the following best represents the decision the youth court will take on allocation?

Select one alternative:

The court must send the matter to the Crown Court for trial as it would be in the interests of justice to do so

The court must retain jurisdiction and can later commit the matter to the Crown Court for sentence if it takes the view after conviction that the likely sentence will be 2 years or more.

The court will send the matter to the Crown Court for trial as their sentencing powers are insufficient to deal with this matter.

The court will send the matter to the Crown Court for trial as the offence is a grave crime.

The court will send the matter to the Crown Court for trial as a sentence substantially beyond their maximum 2 year sentencing powers is a realistic possibility

A
  • The court will send the matter to the Crown Court for trial as a sentence substantially beyond their maximum 2 year sentencing powers is a realistic possibility

This is correct. This answer reflects the test for whether grave crimes should be sent to the CC and the sentencing guidelines suggest a realistic possibility of a sentence substantially beyond 2 years custody; the IOJ test is not used where a youth is charged alone; the court could retain jurisdiction and commit later; it is not obliged to.

35
Q

A defendant at the Crown court is unhappy with her conviction and sentence. She feels that she was wrongly convicted of GBH contrary to s18 OAPA 1861. The only evidence was from her ex-partner who was the victim and only witness to the crime.
He had been choking her and the first thing she could find was a hot iron which she hit him with. She accepts causing a burn and broken tooth, but she does not accept that she did so intentionally as she was acting in self-defence.
What is the ground to be relied upon when appealing the conviction to the Court of Appeal?
Select one alternative:

Conviction is unsafe

Conviction is unjust

Misdirection or error of law

Point of law of general importance

Interest of justice

A
  • Conviction is unsafe

This is correct. A point of law of general importance is a consideration on an appeal to the Supreme Court from the Court of Appeal. A conviction may be unfair or unjust but the ground is that it must be unsafe. Likewise, the conviction can be considered an error of law or a misdirection but the ground is that the conviction is unsafe. A conviction is unsafe where there is an error of law, the judge has misdirected the jury or failed to give a relevant direction, there was bias that would have affected a jury’s decision or the judge was wrong in their exercise of discretion to exclude or admit evidence.

36
Q

A client is due to be sentenced for an offence of theft of £100 from her employer. It is alleged that she stole £20 on five separate occasions from the cash register. She has a previous conviction for theft of baby formula from a shop from three months ago. She has told you that she has been struggling financially since her partner walked out on her and her six-month old baby.

What are the aggravating and mitigating factors that the court are likely to consider in this case?

Select one alternative:

Her family situation and financial difficulties are likely to be a mitigating factor, there are no aggravating factors.

Her family situation, financial difficulties, previous convictions and the amount stolen are aggravating factors, there are no mitigating factors.

Her family situation, financial difficulties and previous convictions are likely to be aggravating factors, there are no mitigating factors.

Her family situation and financial difficulties are likely to be a mitigating factor, the previous conviction is likely to be an aggravating factor.
Answered and correct

Her family situation and financial difficulties are likely to be a mitigating factor, the amount stolen is likely to be an aggravating factor.

A
  • Her family situation and financial difficulties are likely to be a mitigating factor, the previous conviction is likely to be an aggravating factor.

The fact that she has a previous conviction is an aggravating factor. The amount of money would not be viewed as an aggravating factor in this scenario. Her financial and family situation would likely be viewed as mitigating factors.

37
Q

You act for a defendant who is charged with a night-time burglary of a dwelling house. Entry appears to have been gained by breaking a glass panel in the kitchen door and personal items were stolen from the bedroom. The occupant was on holiday at the time and there are no eye witnesses. The defendant denies being the person responsible.

The defendant has the following previous convictions.

6 months ago: Criminal Damage - Details: scratch damage to a motor vehicle.

12 months ago: Common Assault - Details: fight after football match

15 months ago: Theft - Details: theft of clothes from department store value £50.

24 months ago: Theft (Dwelling) - Details: Theft of £600 cash whilst lodging at the premises.

4 years ago: Obtaining property by deception -Details: used cloned credit card to purchase goods worth £850.

5 years ago: Burglary of dwelling house - Details: Premises unoccupied and limited damage caused.

The prosecution has notified you that it wishes to adduce certain convictions as evidence of bad character at the defendant’s trial.

What is the most likely course of action the prosecution would to take in relation to an application to admit the defendant’s previous convictions as evidence of bad character?

Select one alternative:

Although the burglary and theft convictions show a propensity to commit offences of the type charged, the court will not admit convictions to bolster an otherwise weak case.

Only the burglary conviction could potentially be admitted to show propensity to commit offences of the type charged but it is too old.

Make an application to adduce the burglary and theft convictions as important explanatory evidence

Make an application to adduce the obtaining property by deception conviction as showing propensity to be untruthful.

Make an application to adduce the burglary and theft convictions to show a propensity to commit offences of the type charged.

A

Although the burglary and theft convictions show a propensity to commit offences of the type charged, the court will not admit convictions to bolster an otherwise weak case.

This is correct. A defendant’s propensity to commit offences of the kind with which he is charged may (without prejudice to any other way of doing so) be established by evidence that he has been convicted of an offence of the same description as the one with which he is charged, or an offence of the same category as the one with which he is charged. The theft and burglary convictions would be covered by this. However, unless there is other evidence the court will be very unlikely to admit the convictions.

38
Q

18
The Defendant appears before the Youth Court and enters a plea of guilty to a robbery of a mobile phone from a pupil at school. The defendant is 15 years old and has never been arrested by the police before. This is the first time they have appeared before a court.
Which of the following best summarises the likely sentence that will be imposed on the defendant?
Select one alternative:

The Youth Court will impose a referral order as this is the first offence and the defendant has entered a plea of guilty.
Answered and correct

The Youth Court will commit the defendant to the Crown Court for sentence as robbery is a grave crime.

The Youth Court will impose a supervision order with ISS as a direct alternative to custody as robbery is a serious offence but the defendant cannot receive a custodial sentence because of their age.

The Youth Court will impose a reparation order as there is an identifiable victim.

The Youth Court will commit the defendant to the Crown Court for sentence as robbery is indictable only.

A
  • The Youth Court will impose a referral order as this is the first offence and the defendant has entered a plea of guilty.

A referral order is the mandatory sentence in a youth court where the youth has committed and offence for the first time and has pleaded guilty to an imprisonable offence. There are exceptions, namely where a custodial sentence or discharge are more appropriate. For the case to be committed as a grave crime there must be a real prospect of the sentence being in excess of two years which will be unlikely here. That robbery is indictable only is not relevant as we are dealing with a youth. As the defendant is aged 15 there are no restrictions on the imposition of a DTO if the custody threshold is met.

39
Q

The defendant is on trial for a serious night-time burglary. The defendant denies being the burglar. As part of the prosecution case the 87 year old occupier of the house provided a statement describing the events of the night in question, how they were disturbed and came downstairs in the morning to find that they had been burgled. They did not see the offender. Sadly, the occupier has since died before the case is listed for trial and the prosecution can produce the death certificate if necessary.

Which of the following best reflects whether the statement of the occupier will be read to the court?
Select one alternative:

The statement will not be read. It is hearsay. Although the fact that the maker is dead is a condition that allows for the statement to be read,the court will not grant leave as it is not in the interests of justice.

The statement will not be read. It is hearsay. Although the fact that the maker is dead is a condition that allows for the statement to be read, the defence would successfully argue that its admission would have an adverse effect on the fairness of the case.

The statement will be read because although it is hearsay the maker is dead and therefore it is prima facie admissible.
Is correct

The statement will not be read as it is inadmissible hearsay. It is being adduced in order to prove the truth of the fact that defendant burgled the occupier’s house.

The statement will be read. It is not hearsay and so is admissible.

A
  • The statement will be read because although it is hearsay the maker is dead and therefore it is prima facie admissible.

The statement is hearsay because it goes towards the truth of what the occupier says: namely that the burglary happened in the way described. It goes towards the truth of what the occupier says but not that the burglar was the defendant. Rather, it is evidence that the burglary happened in the way described. These are not necessarily facts in issue but the statement itself is still hearsay. Given the maker is dead, the statement meets the criteria for admission under the statutory rules. There are no grounds to argue against its admission. The facts are not in issue. There is no prejudice to the defendant as the occupier’s evidence would not add weight to the issue that the defendant was the burglar. [Note – in practice the defence might have agreed to the statement being read in any event under s 9 CJA 1967. This is not an option in this question. Always answer the question based on what it is asking and the options available.]

40
Q

At the plea before venue hearing at the magistrates’ court your client enters a plea of guilty to an offence of theft. At the sentence hearing the magistrates, having heard all of the mitigation, impose a six month custodial sentence. Your client asks you to advise him about an appeal.
Which statement best describes the advice you would give to your client?
Select one alternative:

Your client can appeal his sentence provided he lodges the notice of appeal within 28 days from the date of sentence to the Crown Court.

Your client can appeal against his conviction and sentence provided he lodges a notice of appeal within 15 business days from the date of sentence to the magistrates’ court.
Answered and wrong

Your client has no right to appeal.

Your client can appeal against his sentence as long as detailed grounds of the appeal are set out in the notice of appeal.

Your client can appeal the sentence provided he lodges a notice of appeal within 15 business days from the date of sentence to the magistrates’ court.
Is correct

A

Your client cannot appeal against the conviction because a guilty plea has been entered and therefore the offence has been admitted. However, the defendant can appeal sentence as of right. The notice of appeal does not need detailed grounds. However, it should set out the circumstances of the appeal. The correct procedure is that the notice of appeal must be lodged at the magistrates’ court within 15 business days of the sentence hearing date.

Your client cannot appeal against the conviction because a guilty plea has been entered and therefore the offence has been admitted. However, the defendant can appeal sentence as of right. The notice of appeal does not need detailed grounds. However, it should set out the circumstances of the appeal. The correct procedure is that the notice of appeal must be lodged at the magistrates’ court within 15 business days of the sentence hearing date.